Find last 3 digits of $ 2032^2031^2030^dots^2^1$mod Distributive Law, factoring $!!bmod!!:$ $ abbmod ac = a(bbmod c)$A power-exponential congruence equationHow to calculate $a^b$ (mod 10, 100 etc. ), where $a$ is not co-prime to $10$.Why is $x^100 = 1 mod 1000$ if $x < 1000$ and $gcd (x,1000) = 1$?Last 3 digits of Marsenne numbersShow that $a^varphi(n)+1equiv a,mod n$.Showing that $a^5$ and $a$ have the same last digit using Euler's Theorem.Find the last ten digits of this exponential tower.Euler theorem, finding last digitsRSA decryption coefficientFinding the last 4 digits of a huge power

Must a CPU have a GPU if the motherboard provides a display port (when there isn't any separate video card)?

The best in flight meal option for those suffering from reflux

Is Jesus the last Prophet?

Are athlete's college degrees discounted by employers and graduate school admissions?

Is there a radar system monitoring the UK mainland border?

Placement of positioning lights on A320 winglets

Does WiFi affect the quality of images downloaded from the internet?

What game uses dice with engraved faces, weapon symbols, double weapon symbols and object symbols?

Is it true that "only photographers care about noise"?

ISP is not hashing the password I log in with online. Should I take any action?

Why is linear regression results so much different from Poisson regression?

Can an open source licence be revoked if it violates employer's IP?

As easy as Three, Two, One... How fast can you go from Five to Four?

Why would a car salesman tell me not to get my credit pulled again?

Generate parentheses solution

Part of my house is inexplicably gone

Can I get a photo of an Ancient Arrow?

usage of mir gefallen

Must I use my personal social media account for work?

Why is it bad to use your whole foot in rock climbing

Tiffeneau–Demjanov rearrangement products

What did the 8086 (and 8088) do upon encountering an illegal instruction?

Can a non-diagonal 2x2 matrix with just one eigenvalue be diagonalizable?

In The Incredibles 2, why does Screenslaver's name use a pun on something that doesn't exist in the 1950s pastiche?



Find last 3 digits of $ 2032^2031^2030^dots^2^1$


mod Distributive Law, factoring $!!bmod!!:$ $ abbmod ac = a(bbmod c)$A power-exponential congruence equationHow to calculate $a^b$ (mod 10, 100 etc. ), where $a$ is not co-prime to $10$.Why is $x^100 = 1 mod 1000$ if $x < 1000$ and $gcd (x,1000) = 1$?Last 3 digits of Marsenne numbersShow that $a^varphi(n)+1equiv a,mod n$.Showing that $a^5$ and $a$ have the same last digit using Euler's Theorem.Find the last ten digits of this exponential tower.Euler theorem, finding last digitsRSA decryption coefficientFinding the last 4 digits of a huge power













7












$begingroup$


Find the last 3 digits of this number
$$
2032^2031^2030^dots^2^1
$$

So obviously we are looking for $x$ so that
$$
2032^2031^2030^dots^2^1 equiv x quad textmodhspace0.1cm 1000
$$

I also know that usually you use Euler' theorem here, but that only works when the numbers $a$ and $n$ are coprime, but $2032$ and $1000$ are not coprime? I can easily find $varphi(1000)$, that is not a problem. Am I looking for wrong numbers to be coprime here or is there another way instead of Euler' theorem?










share|cite|improve this question











$endgroup$







  • 2




    $begingroup$
    The common factor here is $8$ and you should be able show that he power is divisible by $8$, so the residue modulo $1000$ can be determined by looking at the residue modulo $1000/8=125$
    $endgroup$
    – Mark Bennet
    Mar 24 at 21:20










  • $begingroup$
    @Mark is correct, and in fact we can factor it out in a slick operational way that avoids using CRT by using the mod Distributive Law, as I show in my answer. This usually ends up being simpler than rotely applying CRT = Chinese Remainder when the base shares a common factor with the modulus.
    $endgroup$
    – Bill Dubuque
    Mar 24 at 23:59











  • $begingroup$
    Bah, that's no monster. Graham's Number is a monster!
    $endgroup$
    – Cort Ammon
    Mar 25 at 5:48















7












$begingroup$


Find the last 3 digits of this number
$$
2032^2031^2030^dots^2^1
$$

So obviously we are looking for $x$ so that
$$
2032^2031^2030^dots^2^1 equiv x quad textmodhspace0.1cm 1000
$$

I also know that usually you use Euler' theorem here, but that only works when the numbers $a$ and $n$ are coprime, but $2032$ and $1000$ are not coprime? I can easily find $varphi(1000)$, that is not a problem. Am I looking for wrong numbers to be coprime here or is there another way instead of Euler' theorem?










share|cite|improve this question











$endgroup$







  • 2




    $begingroup$
    The common factor here is $8$ and you should be able show that he power is divisible by $8$, so the residue modulo $1000$ can be determined by looking at the residue modulo $1000/8=125$
    $endgroup$
    – Mark Bennet
    Mar 24 at 21:20










  • $begingroup$
    @Mark is correct, and in fact we can factor it out in a slick operational way that avoids using CRT by using the mod Distributive Law, as I show in my answer. This usually ends up being simpler than rotely applying CRT = Chinese Remainder when the base shares a common factor with the modulus.
    $endgroup$
    – Bill Dubuque
    Mar 24 at 23:59











  • $begingroup$
    Bah, that's no monster. Graham's Number is a monster!
    $endgroup$
    – Cort Ammon
    Mar 25 at 5:48













7












7








7


2



$begingroup$


Find the last 3 digits of this number
$$
2032^2031^2030^dots^2^1
$$

So obviously we are looking for $x$ so that
$$
2032^2031^2030^dots^2^1 equiv x quad textmodhspace0.1cm 1000
$$

I also know that usually you use Euler' theorem here, but that only works when the numbers $a$ and $n$ are coprime, but $2032$ and $1000$ are not coprime? I can easily find $varphi(1000)$, that is not a problem. Am I looking for wrong numbers to be coprime here or is there another way instead of Euler' theorem?










share|cite|improve this question











$endgroup$




Find the last 3 digits of this number
$$
2032^2031^2030^dots^2^1
$$

So obviously we are looking for $x$ so that
$$
2032^2031^2030^dots^2^1 equiv x quad textmodhspace0.1cm 1000
$$

I also know that usually you use Euler' theorem here, but that only works when the numbers $a$ and $n$ are coprime, but $2032$ and $1000$ are not coprime? I can easily find $varphi(1000)$, that is not a problem. Am I looking for wrong numbers to be coprime here or is there another way instead of Euler' theorem?







number-theory totient-function






share|cite|improve this question















share|cite|improve this question













share|cite|improve this question




share|cite|improve this question








edited Mar 25 at 7:49









Asaf Karagila

312k33446780




312k33446780










asked Mar 24 at 20:59









Kristin PeterselKristin Petersel

1056




1056







  • 2




    $begingroup$
    The common factor here is $8$ and you should be able show that he power is divisible by $8$, so the residue modulo $1000$ can be determined by looking at the residue modulo $1000/8=125$
    $endgroup$
    – Mark Bennet
    Mar 24 at 21:20










  • $begingroup$
    @Mark is correct, and in fact we can factor it out in a slick operational way that avoids using CRT by using the mod Distributive Law, as I show in my answer. This usually ends up being simpler than rotely applying CRT = Chinese Remainder when the base shares a common factor with the modulus.
    $endgroup$
    – Bill Dubuque
    Mar 24 at 23:59











  • $begingroup$
    Bah, that's no monster. Graham's Number is a monster!
    $endgroup$
    – Cort Ammon
    Mar 25 at 5:48












  • 2




    $begingroup$
    The common factor here is $8$ and you should be able show that he power is divisible by $8$, so the residue modulo $1000$ can be determined by looking at the residue modulo $1000/8=125$
    $endgroup$
    – Mark Bennet
    Mar 24 at 21:20










  • $begingroup$
    @Mark is correct, and in fact we can factor it out in a slick operational way that avoids using CRT by using the mod Distributive Law, as I show in my answer. This usually ends up being simpler than rotely applying CRT = Chinese Remainder when the base shares a common factor with the modulus.
    $endgroup$
    – Bill Dubuque
    Mar 24 at 23:59











  • $begingroup$
    Bah, that's no monster. Graham's Number is a monster!
    $endgroup$
    – Cort Ammon
    Mar 25 at 5:48







2




2




$begingroup$
The common factor here is $8$ and you should be able show that he power is divisible by $8$, so the residue modulo $1000$ can be determined by looking at the residue modulo $1000/8=125$
$endgroup$
– Mark Bennet
Mar 24 at 21:20




$begingroup$
The common factor here is $8$ and you should be able show that he power is divisible by $8$, so the residue modulo $1000$ can be determined by looking at the residue modulo $1000/8=125$
$endgroup$
– Mark Bennet
Mar 24 at 21:20












$begingroup$
@Mark is correct, and in fact we can factor it out in a slick operational way that avoids using CRT by using the mod Distributive Law, as I show in my answer. This usually ends up being simpler than rotely applying CRT = Chinese Remainder when the base shares a common factor with the modulus.
$endgroup$
– Bill Dubuque
Mar 24 at 23:59





$begingroup$
@Mark is correct, and in fact we can factor it out in a slick operational way that avoids using CRT by using the mod Distributive Law, as I show in my answer. This usually ends up being simpler than rotely applying CRT = Chinese Remainder when the base shares a common factor with the modulus.
$endgroup$
– Bill Dubuque
Mar 24 at 23:59













$begingroup$
Bah, that's no monster. Graham's Number is a monster!
$endgroup$
– Cort Ammon
Mar 25 at 5:48




$begingroup$
Bah, that's no monster. Graham's Number is a monster!
$endgroup$
– Cort Ammon
Mar 25 at 5:48










4 Answers
4






active

oldest

votes


















7












$begingroup$

$bmod 1000!: 32^large 2031^LARGE 2k!!equiv, 8left[dfraccolor#0a032^large 2031^LARGE 2k8 bmod color#0a0125right]! equiv 8left[dfraccolor#c00328bmod 125right]! equiv 32, $ by



$ ,beginalign !bmod color#0a0125!: color#0a032^large 2031^LARGE 2k!!
&equiv, 2^large 5cdot 2031^LARGE 2k! bmod 100 rm by 100 = phi(125) rm [Euler totient]\
&equiv,2^large 5(color#b6f2031^LARGE color#d4f2k! bmod 20) rm by mod Distributive Law\
&equiv,2^large 5(color#b6f1^LARGE k)equiv, color#c0032 rm by color#b6f2031^large 2!equiv 11^large 2equivcolor#b6f 1!!!pmod!20\
endalign $






share|cite|improve this answer











$endgroup$








  • 2




    $begingroup$
    We used twice: $, abbmod ac, =, a,(bbmod c), =, $ mod Distributive Law $ $
    $endgroup$
    – Bill Dubuque
    Mar 24 at 22:50











  • $begingroup$
    How would you prove this?
    $endgroup$
    – Markus Punnar
    Mar 25 at 11:23










  • $begingroup$
    @Markus Prove what? If you mean the mod Distributive Law then follow the link in my prior comment.
    $endgroup$
    – Bill Dubuque
    Mar 25 at 13:05


















4












$begingroup$

It's a lot simpler than it looks. I shall call the number $N$.



You will know the residue modulo $10^3$, thus the last three digits, if you first get the residues modulo $2^3=8$ and modulo $5^3=125$.



$N$ is obviously a multiple of $8$, thus $Nequiv 0bmod 8$. Which leaves $bmod 125$.



The base $2032equiv 32$. When this is raised to a power, the residue of this power depends only on the residue of the exponent $bmod 100$ where $100$ is the Euler totient of $125$. But the exponent on $2032$ has the form



$2031^10k=(2030+1)^10k=(textbinomial expansion)=100m+1$



So $Nequiv 32^1equiv 32bmod 125$. The only multiple of $8$ between $0$ and $999$ satisfying this result is $32$ itself so ... $Nequiv 32bmod 1000$. Meaning the last three digits were there all along, the $colorblue032$ in the base $2032$!






share|cite|improve this answer











$endgroup$








  • 2




    $begingroup$
    (+1) same answer I got, by means similar enough that I won't add another post here.
    $endgroup$
    – robjohn
    Mar 24 at 22:25











  • $begingroup$
    Worth emphasis is that arguments like this can be presented completely operationally by employing the mod Distributive Law, and this clarifies and simplifies the arithmetic - see my answer.
    $endgroup$
    – Bill Dubuque
    Mar 24 at 23:27



















3












$begingroup$

Don't be scared. If it turns into a monster and eats you, run away after you throw stones at it. Don't run away before throwing stones just because it looks like a monster.



$2032^monster$ and $1000$ are relatively prime so we can't use Euler theorem but we can break it down with Chinese remainder theorem.



$2032^monster = 0 pmod 8$ and so we just need to solve $2032^monster pmod 125$ and for that we can use Euler Theorem.



$phi(125=5^3) = (5-1)*5^3-1 = 100$.



So $2032^monster equiv 32^monster % 100$.



And $monster = 2031^littlemonsterequiv 31^littlemonsterpmod 100$



$31$ and $100$ are relatively prime and $phi(100)= 40$ so



$31^littlemonster equiv 31^littlemonster % 40 pmod 100$.



$littlemonster = 2030^smallmonster$ but $5|2030$ and as $smallmonster > 2$ we know $8|2^smallmonster$ and $2^smallmonster|2030^smallmonster$.



So $littlemonster equiv 0 pmod 40$.



$2031^littlemonster equiv 31^0 equiv 1 pmod 100$



So $2032^monster equiv 32 pmod 125$



So $2032^monster equiv 0 pmod 8$ and $2032^monster equiv 32 pmod 125$.



As $8|32$ we are done. $2032^monster equiv 32 pmod 1000$.



and the last three digits are $032$.






share|cite|improve this answer











$endgroup$








  • 1




    $begingroup$
    Good idea, but you rendered $2031^2030^··· bmod 125$. You need $2032^2031^2030^··· bmod 125$.
    $endgroup$
    – Oscar Lanzi
    Mar 25 at 1:30










  • $begingroup$
    oops..............
    $endgroup$
    – fleablood
    Mar 25 at 11:58






  • 1




    $begingroup$
    Actually I rendered it as $2032^2031^...equiv 31^2031^...pmod125$... which is still an error.
    $endgroup$
    – fleablood
    Mar 25 at 12:02


















2












$begingroup$

By the Chinese Remainder Theorem, if you want to find what remainder a given number has when divided by $1000$, you can split that into 2 problems: Find the remainder$mod 8$ and$mod 125$. Obviously



$$z_0:=2032^2031^2030^dots^2^1 equiv 0 pmod 8$$



What remains to be found is $x_0 in [0,124]$ in



$$z_0 equiv x_0 pmod 125.$$



As $z_0$ is now coprime to $125$, you can apply Euler's theorem now. With



$$z_1:=2031^2030^dots^2^1$$



and



$$phi(125)=100$$ the new problem becomes to find $x_1 in [0,99]$ in



$$z_1 equiv x_1 pmod 100$$



and then use



$$32^x_1 equiv x_0 pmod 125$$



to find $x_0$.



So this reduced the original problem$mod 1000$ to a smaller problem$mod 100$.



Applying this reduction procedure a few more times (using the Chinese Remainder Theorem if appropriate), should result in congruences with smaller and smaller module that can in the end be solved (e.g $mod 2$).



Then to solve the original problem you need to back-substitute the calculated $x_i$ to get $x_i-1$, just as outlined for $x_1,x_0$ above.






share|cite|improve this answer









$endgroup$













    Your Answer








    StackExchange.ready(function()
    var channelOptions =
    tags: "".split(" "),
    id: "69"
    ;
    initTagRenderer("".split(" "), "".split(" "), channelOptions);

    StackExchange.using("externalEditor", function()
    // Have to fire editor after snippets, if snippets enabled
    if (StackExchange.settings.snippets.snippetsEnabled)
    StackExchange.using("snippets", function()
    createEditor();
    );

    else
    createEditor();

    );

    function createEditor()
    StackExchange.prepareEditor(
    heartbeatType: 'answer',
    autoActivateHeartbeat: false,
    convertImagesToLinks: true,
    noModals: true,
    showLowRepImageUploadWarning: true,
    reputationToPostImages: 10,
    bindNavPrevention: true,
    postfix: "",
    imageUploader:
    brandingHtml: "Powered by u003ca class="icon-imgur-white" href="https://imgur.com/"u003eu003c/au003e",
    contentPolicyHtml: "User contributions licensed under u003ca href="https://creativecommons.org/licenses/by-sa/3.0/"u003ecc by-sa 3.0 with attribution requiredu003c/au003e u003ca href="https://stackoverflow.com/legal/content-policy"u003e(content policy)u003c/au003e",
    allowUrls: true
    ,
    noCode: true, onDemand: true,
    discardSelector: ".discard-answer"
    ,immediatelyShowMarkdownHelp:true
    );



    );













    draft saved

    draft discarded


















    StackExchange.ready(
    function ()
    StackExchange.openid.initPostLogin('.new-post-login', 'https%3a%2f%2fmath.stackexchange.com%2fquestions%2f3161051%2ffind-last-3-digits-of-203220312030-dots21%23new-answer', 'question_page');

    );

    Post as a guest















    Required, but never shown

























    4 Answers
    4






    active

    oldest

    votes








    4 Answers
    4






    active

    oldest

    votes









    active

    oldest

    votes






    active

    oldest

    votes









    7












    $begingroup$

    $bmod 1000!: 32^large 2031^LARGE 2k!!equiv, 8left[dfraccolor#0a032^large 2031^LARGE 2k8 bmod color#0a0125right]! equiv 8left[dfraccolor#c00328bmod 125right]! equiv 32, $ by



    $ ,beginalign !bmod color#0a0125!: color#0a032^large 2031^LARGE 2k!!
    &equiv, 2^large 5cdot 2031^LARGE 2k! bmod 100 rm by 100 = phi(125) rm [Euler totient]\
    &equiv,2^large 5(color#b6f2031^LARGE color#d4f2k! bmod 20) rm by mod Distributive Law\
    &equiv,2^large 5(color#b6f1^LARGE k)equiv, color#c0032 rm by color#b6f2031^large 2!equiv 11^large 2equivcolor#b6f 1!!!pmod!20\
    endalign $






    share|cite|improve this answer











    $endgroup$








    • 2




      $begingroup$
      We used twice: $, abbmod ac, =, a,(bbmod c), =, $ mod Distributive Law $ $
      $endgroup$
      – Bill Dubuque
      Mar 24 at 22:50











    • $begingroup$
      How would you prove this?
      $endgroup$
      – Markus Punnar
      Mar 25 at 11:23










    • $begingroup$
      @Markus Prove what? If you mean the mod Distributive Law then follow the link in my prior comment.
      $endgroup$
      – Bill Dubuque
      Mar 25 at 13:05















    7












    $begingroup$

    $bmod 1000!: 32^large 2031^LARGE 2k!!equiv, 8left[dfraccolor#0a032^large 2031^LARGE 2k8 bmod color#0a0125right]! equiv 8left[dfraccolor#c00328bmod 125right]! equiv 32, $ by



    $ ,beginalign !bmod color#0a0125!: color#0a032^large 2031^LARGE 2k!!
    &equiv, 2^large 5cdot 2031^LARGE 2k! bmod 100 rm by 100 = phi(125) rm [Euler totient]\
    &equiv,2^large 5(color#b6f2031^LARGE color#d4f2k! bmod 20) rm by mod Distributive Law\
    &equiv,2^large 5(color#b6f1^LARGE k)equiv, color#c0032 rm by color#b6f2031^large 2!equiv 11^large 2equivcolor#b6f 1!!!pmod!20\
    endalign $






    share|cite|improve this answer











    $endgroup$








    • 2




      $begingroup$
      We used twice: $, abbmod ac, =, a,(bbmod c), =, $ mod Distributive Law $ $
      $endgroup$
      – Bill Dubuque
      Mar 24 at 22:50











    • $begingroup$
      How would you prove this?
      $endgroup$
      – Markus Punnar
      Mar 25 at 11:23










    • $begingroup$
      @Markus Prove what? If you mean the mod Distributive Law then follow the link in my prior comment.
      $endgroup$
      – Bill Dubuque
      Mar 25 at 13:05













    7












    7








    7





    $begingroup$

    $bmod 1000!: 32^large 2031^LARGE 2k!!equiv, 8left[dfraccolor#0a032^large 2031^LARGE 2k8 bmod color#0a0125right]! equiv 8left[dfraccolor#c00328bmod 125right]! equiv 32, $ by



    $ ,beginalign !bmod color#0a0125!: color#0a032^large 2031^LARGE 2k!!
    &equiv, 2^large 5cdot 2031^LARGE 2k! bmod 100 rm by 100 = phi(125) rm [Euler totient]\
    &equiv,2^large 5(color#b6f2031^LARGE color#d4f2k! bmod 20) rm by mod Distributive Law\
    &equiv,2^large 5(color#b6f1^LARGE k)equiv, color#c0032 rm by color#b6f2031^large 2!equiv 11^large 2equivcolor#b6f 1!!!pmod!20\
    endalign $






    share|cite|improve this answer











    $endgroup$



    $bmod 1000!: 32^large 2031^LARGE 2k!!equiv, 8left[dfraccolor#0a032^large 2031^LARGE 2k8 bmod color#0a0125right]! equiv 8left[dfraccolor#c00328bmod 125right]! equiv 32, $ by



    $ ,beginalign !bmod color#0a0125!: color#0a032^large 2031^LARGE 2k!!
    &equiv, 2^large 5cdot 2031^LARGE 2k! bmod 100 rm by 100 = phi(125) rm [Euler totient]\
    &equiv,2^large 5(color#b6f2031^LARGE color#d4f2k! bmod 20) rm by mod Distributive Law\
    &equiv,2^large 5(color#b6f1^LARGE k)equiv, color#c0032 rm by color#b6f2031^large 2!equiv 11^large 2equivcolor#b6f 1!!!pmod!20\
    endalign $







    share|cite|improve this answer














    share|cite|improve this answer



    share|cite|improve this answer








    edited Mar 24 at 23:40

























    answered Mar 24 at 22:48









    Bill DubuqueBill Dubuque

    217k29206668




    217k29206668







    • 2




      $begingroup$
      We used twice: $, abbmod ac, =, a,(bbmod c), =, $ mod Distributive Law $ $
      $endgroup$
      – Bill Dubuque
      Mar 24 at 22:50











    • $begingroup$
      How would you prove this?
      $endgroup$
      – Markus Punnar
      Mar 25 at 11:23










    • $begingroup$
      @Markus Prove what? If you mean the mod Distributive Law then follow the link in my prior comment.
      $endgroup$
      – Bill Dubuque
      Mar 25 at 13:05












    • 2




      $begingroup$
      We used twice: $, abbmod ac, =, a,(bbmod c), =, $ mod Distributive Law $ $
      $endgroup$
      – Bill Dubuque
      Mar 24 at 22:50











    • $begingroup$
      How would you prove this?
      $endgroup$
      – Markus Punnar
      Mar 25 at 11:23










    • $begingroup$
      @Markus Prove what? If you mean the mod Distributive Law then follow the link in my prior comment.
      $endgroup$
      – Bill Dubuque
      Mar 25 at 13:05







    2




    2




    $begingroup$
    We used twice: $, abbmod ac, =, a,(bbmod c), =, $ mod Distributive Law $ $
    $endgroup$
    – Bill Dubuque
    Mar 24 at 22:50





    $begingroup$
    We used twice: $, abbmod ac, =, a,(bbmod c), =, $ mod Distributive Law $ $
    $endgroup$
    – Bill Dubuque
    Mar 24 at 22:50













    $begingroup$
    How would you prove this?
    $endgroup$
    – Markus Punnar
    Mar 25 at 11:23




    $begingroup$
    How would you prove this?
    $endgroup$
    – Markus Punnar
    Mar 25 at 11:23












    $begingroup$
    @Markus Prove what? If you mean the mod Distributive Law then follow the link in my prior comment.
    $endgroup$
    – Bill Dubuque
    Mar 25 at 13:05




    $begingroup$
    @Markus Prove what? If you mean the mod Distributive Law then follow the link in my prior comment.
    $endgroup$
    – Bill Dubuque
    Mar 25 at 13:05











    4












    $begingroup$

    It's a lot simpler than it looks. I shall call the number $N$.



    You will know the residue modulo $10^3$, thus the last three digits, if you first get the residues modulo $2^3=8$ and modulo $5^3=125$.



    $N$ is obviously a multiple of $8$, thus $Nequiv 0bmod 8$. Which leaves $bmod 125$.



    The base $2032equiv 32$. When this is raised to a power, the residue of this power depends only on the residue of the exponent $bmod 100$ where $100$ is the Euler totient of $125$. But the exponent on $2032$ has the form



    $2031^10k=(2030+1)^10k=(textbinomial expansion)=100m+1$



    So $Nequiv 32^1equiv 32bmod 125$. The only multiple of $8$ between $0$ and $999$ satisfying this result is $32$ itself so ... $Nequiv 32bmod 1000$. Meaning the last three digits were there all along, the $colorblue032$ in the base $2032$!






    share|cite|improve this answer











    $endgroup$








    • 2




      $begingroup$
      (+1) same answer I got, by means similar enough that I won't add another post here.
      $endgroup$
      – robjohn
      Mar 24 at 22:25











    • $begingroup$
      Worth emphasis is that arguments like this can be presented completely operationally by employing the mod Distributive Law, and this clarifies and simplifies the arithmetic - see my answer.
      $endgroup$
      – Bill Dubuque
      Mar 24 at 23:27
















    4












    $begingroup$

    It's a lot simpler than it looks. I shall call the number $N$.



    You will know the residue modulo $10^3$, thus the last three digits, if you first get the residues modulo $2^3=8$ and modulo $5^3=125$.



    $N$ is obviously a multiple of $8$, thus $Nequiv 0bmod 8$. Which leaves $bmod 125$.



    The base $2032equiv 32$. When this is raised to a power, the residue of this power depends only on the residue of the exponent $bmod 100$ where $100$ is the Euler totient of $125$. But the exponent on $2032$ has the form



    $2031^10k=(2030+1)^10k=(textbinomial expansion)=100m+1$



    So $Nequiv 32^1equiv 32bmod 125$. The only multiple of $8$ between $0$ and $999$ satisfying this result is $32$ itself so ... $Nequiv 32bmod 1000$. Meaning the last three digits were there all along, the $colorblue032$ in the base $2032$!






    share|cite|improve this answer











    $endgroup$








    • 2




      $begingroup$
      (+1) same answer I got, by means similar enough that I won't add another post here.
      $endgroup$
      – robjohn
      Mar 24 at 22:25











    • $begingroup$
      Worth emphasis is that arguments like this can be presented completely operationally by employing the mod Distributive Law, and this clarifies and simplifies the arithmetic - see my answer.
      $endgroup$
      – Bill Dubuque
      Mar 24 at 23:27














    4












    4








    4





    $begingroup$

    It's a lot simpler than it looks. I shall call the number $N$.



    You will know the residue modulo $10^3$, thus the last three digits, if you first get the residues modulo $2^3=8$ and modulo $5^3=125$.



    $N$ is obviously a multiple of $8$, thus $Nequiv 0bmod 8$. Which leaves $bmod 125$.



    The base $2032equiv 32$. When this is raised to a power, the residue of this power depends only on the residue of the exponent $bmod 100$ where $100$ is the Euler totient of $125$. But the exponent on $2032$ has the form



    $2031^10k=(2030+1)^10k=(textbinomial expansion)=100m+1$



    So $Nequiv 32^1equiv 32bmod 125$. The only multiple of $8$ between $0$ and $999$ satisfying this result is $32$ itself so ... $Nequiv 32bmod 1000$. Meaning the last three digits were there all along, the $colorblue032$ in the base $2032$!






    share|cite|improve this answer











    $endgroup$



    It's a lot simpler than it looks. I shall call the number $N$.



    You will know the residue modulo $10^3$, thus the last three digits, if you first get the residues modulo $2^3=8$ and modulo $5^3=125$.



    $N$ is obviously a multiple of $8$, thus $Nequiv 0bmod 8$. Which leaves $bmod 125$.



    The base $2032equiv 32$. When this is raised to a power, the residue of this power depends only on the residue of the exponent $bmod 100$ where $100$ is the Euler totient of $125$. But the exponent on $2032$ has the form



    $2031^10k=(2030+1)^10k=(textbinomial expansion)=100m+1$



    So $Nequiv 32^1equiv 32bmod 125$. The only multiple of $8$ between $0$ and $999$ satisfying this result is $32$ itself so ... $Nequiv 32bmod 1000$. Meaning the last three digits were there all along, the $colorblue032$ in the base $2032$!







    share|cite|improve this answer














    share|cite|improve this answer



    share|cite|improve this answer








    edited Mar 24 at 21:36

























    answered Mar 24 at 21:30









    Oscar LanziOscar Lanzi

    14.1k22137




    14.1k22137







    • 2




      $begingroup$
      (+1) same answer I got, by means similar enough that I won't add another post here.
      $endgroup$
      – robjohn
      Mar 24 at 22:25











    • $begingroup$
      Worth emphasis is that arguments like this can be presented completely operationally by employing the mod Distributive Law, and this clarifies and simplifies the arithmetic - see my answer.
      $endgroup$
      – Bill Dubuque
      Mar 24 at 23:27













    • 2




      $begingroup$
      (+1) same answer I got, by means similar enough that I won't add another post here.
      $endgroup$
      – robjohn
      Mar 24 at 22:25











    • $begingroup$
      Worth emphasis is that arguments like this can be presented completely operationally by employing the mod Distributive Law, and this clarifies and simplifies the arithmetic - see my answer.
      $endgroup$
      – Bill Dubuque
      Mar 24 at 23:27








    2




    2




    $begingroup$
    (+1) same answer I got, by means similar enough that I won't add another post here.
    $endgroup$
    – robjohn
    Mar 24 at 22:25





    $begingroup$
    (+1) same answer I got, by means similar enough that I won't add another post here.
    $endgroup$
    – robjohn
    Mar 24 at 22:25













    $begingroup$
    Worth emphasis is that arguments like this can be presented completely operationally by employing the mod Distributive Law, and this clarifies and simplifies the arithmetic - see my answer.
    $endgroup$
    – Bill Dubuque
    Mar 24 at 23:27





    $begingroup$
    Worth emphasis is that arguments like this can be presented completely operationally by employing the mod Distributive Law, and this clarifies and simplifies the arithmetic - see my answer.
    $endgroup$
    – Bill Dubuque
    Mar 24 at 23:27












    3












    $begingroup$

    Don't be scared. If it turns into a monster and eats you, run away after you throw stones at it. Don't run away before throwing stones just because it looks like a monster.



    $2032^monster$ and $1000$ are relatively prime so we can't use Euler theorem but we can break it down with Chinese remainder theorem.



    $2032^monster = 0 pmod 8$ and so we just need to solve $2032^monster pmod 125$ and for that we can use Euler Theorem.



    $phi(125=5^3) = (5-1)*5^3-1 = 100$.



    So $2032^monster equiv 32^monster % 100$.



    And $monster = 2031^littlemonsterequiv 31^littlemonsterpmod 100$



    $31$ and $100$ are relatively prime and $phi(100)= 40$ so



    $31^littlemonster equiv 31^littlemonster % 40 pmod 100$.



    $littlemonster = 2030^smallmonster$ but $5|2030$ and as $smallmonster > 2$ we know $8|2^smallmonster$ and $2^smallmonster|2030^smallmonster$.



    So $littlemonster equiv 0 pmod 40$.



    $2031^littlemonster equiv 31^0 equiv 1 pmod 100$



    So $2032^monster equiv 32 pmod 125$



    So $2032^monster equiv 0 pmod 8$ and $2032^monster equiv 32 pmod 125$.



    As $8|32$ we are done. $2032^monster equiv 32 pmod 1000$.



    and the last three digits are $032$.






    share|cite|improve this answer











    $endgroup$








    • 1




      $begingroup$
      Good idea, but you rendered $2031^2030^··· bmod 125$. You need $2032^2031^2030^··· bmod 125$.
      $endgroup$
      – Oscar Lanzi
      Mar 25 at 1:30










    • $begingroup$
      oops..............
      $endgroup$
      – fleablood
      Mar 25 at 11:58






    • 1




      $begingroup$
      Actually I rendered it as $2032^2031^...equiv 31^2031^...pmod125$... which is still an error.
      $endgroup$
      – fleablood
      Mar 25 at 12:02















    3












    $begingroup$

    Don't be scared. If it turns into a monster and eats you, run away after you throw stones at it. Don't run away before throwing stones just because it looks like a monster.



    $2032^monster$ and $1000$ are relatively prime so we can't use Euler theorem but we can break it down with Chinese remainder theorem.



    $2032^monster = 0 pmod 8$ and so we just need to solve $2032^monster pmod 125$ and for that we can use Euler Theorem.



    $phi(125=5^3) = (5-1)*5^3-1 = 100$.



    So $2032^monster equiv 32^monster % 100$.



    And $monster = 2031^littlemonsterequiv 31^littlemonsterpmod 100$



    $31$ and $100$ are relatively prime and $phi(100)= 40$ so



    $31^littlemonster equiv 31^littlemonster % 40 pmod 100$.



    $littlemonster = 2030^smallmonster$ but $5|2030$ and as $smallmonster > 2$ we know $8|2^smallmonster$ and $2^smallmonster|2030^smallmonster$.



    So $littlemonster equiv 0 pmod 40$.



    $2031^littlemonster equiv 31^0 equiv 1 pmod 100$



    So $2032^monster equiv 32 pmod 125$



    So $2032^monster equiv 0 pmod 8$ and $2032^monster equiv 32 pmod 125$.



    As $8|32$ we are done. $2032^monster equiv 32 pmod 1000$.



    and the last three digits are $032$.






    share|cite|improve this answer











    $endgroup$








    • 1




      $begingroup$
      Good idea, but you rendered $2031^2030^··· bmod 125$. You need $2032^2031^2030^··· bmod 125$.
      $endgroup$
      – Oscar Lanzi
      Mar 25 at 1:30










    • $begingroup$
      oops..............
      $endgroup$
      – fleablood
      Mar 25 at 11:58






    • 1




      $begingroup$
      Actually I rendered it as $2032^2031^...equiv 31^2031^...pmod125$... which is still an error.
      $endgroup$
      – fleablood
      Mar 25 at 12:02













    3












    3








    3





    $begingroup$

    Don't be scared. If it turns into a monster and eats you, run away after you throw stones at it. Don't run away before throwing stones just because it looks like a monster.



    $2032^monster$ and $1000$ are relatively prime so we can't use Euler theorem but we can break it down with Chinese remainder theorem.



    $2032^monster = 0 pmod 8$ and so we just need to solve $2032^monster pmod 125$ and for that we can use Euler Theorem.



    $phi(125=5^3) = (5-1)*5^3-1 = 100$.



    So $2032^monster equiv 32^monster % 100$.



    And $monster = 2031^littlemonsterequiv 31^littlemonsterpmod 100$



    $31$ and $100$ are relatively prime and $phi(100)= 40$ so



    $31^littlemonster equiv 31^littlemonster % 40 pmod 100$.



    $littlemonster = 2030^smallmonster$ but $5|2030$ and as $smallmonster > 2$ we know $8|2^smallmonster$ and $2^smallmonster|2030^smallmonster$.



    So $littlemonster equiv 0 pmod 40$.



    $2031^littlemonster equiv 31^0 equiv 1 pmod 100$



    So $2032^monster equiv 32 pmod 125$



    So $2032^monster equiv 0 pmod 8$ and $2032^monster equiv 32 pmod 125$.



    As $8|32$ we are done. $2032^monster equiv 32 pmod 1000$.



    and the last three digits are $032$.






    share|cite|improve this answer











    $endgroup$



    Don't be scared. If it turns into a monster and eats you, run away after you throw stones at it. Don't run away before throwing stones just because it looks like a monster.



    $2032^monster$ and $1000$ are relatively prime so we can't use Euler theorem but we can break it down with Chinese remainder theorem.



    $2032^monster = 0 pmod 8$ and so we just need to solve $2032^monster pmod 125$ and for that we can use Euler Theorem.



    $phi(125=5^3) = (5-1)*5^3-1 = 100$.



    So $2032^monster equiv 32^monster % 100$.



    And $monster = 2031^littlemonsterequiv 31^littlemonsterpmod 100$



    $31$ and $100$ are relatively prime and $phi(100)= 40$ so



    $31^littlemonster equiv 31^littlemonster % 40 pmod 100$.



    $littlemonster = 2030^smallmonster$ but $5|2030$ and as $smallmonster > 2$ we know $8|2^smallmonster$ and $2^smallmonster|2030^smallmonster$.



    So $littlemonster equiv 0 pmod 40$.



    $2031^littlemonster equiv 31^0 equiv 1 pmod 100$



    So $2032^monster equiv 32 pmod 125$



    So $2032^monster equiv 0 pmod 8$ and $2032^monster equiv 32 pmod 125$.



    As $8|32$ we are done. $2032^monster equiv 32 pmod 1000$.



    and the last three digits are $032$.







    share|cite|improve this answer














    share|cite|improve this answer



    share|cite|improve this answer








    edited Mar 25 at 12:00

























    answered Mar 24 at 22:56









    fleabloodfleablood

    74.4k22892




    74.4k22892







    • 1




      $begingroup$
      Good idea, but you rendered $2031^2030^··· bmod 125$. You need $2032^2031^2030^··· bmod 125$.
      $endgroup$
      – Oscar Lanzi
      Mar 25 at 1:30










    • $begingroup$
      oops..............
      $endgroup$
      – fleablood
      Mar 25 at 11:58






    • 1




      $begingroup$
      Actually I rendered it as $2032^2031^...equiv 31^2031^...pmod125$... which is still an error.
      $endgroup$
      – fleablood
      Mar 25 at 12:02












    • 1




      $begingroup$
      Good idea, but you rendered $2031^2030^··· bmod 125$. You need $2032^2031^2030^··· bmod 125$.
      $endgroup$
      – Oscar Lanzi
      Mar 25 at 1:30










    • $begingroup$
      oops..............
      $endgroup$
      – fleablood
      Mar 25 at 11:58






    • 1




      $begingroup$
      Actually I rendered it as $2032^2031^...equiv 31^2031^...pmod125$... which is still an error.
      $endgroup$
      – fleablood
      Mar 25 at 12:02







    1




    1




    $begingroup$
    Good idea, but you rendered $2031^2030^··· bmod 125$. You need $2032^2031^2030^··· bmod 125$.
    $endgroup$
    – Oscar Lanzi
    Mar 25 at 1:30




    $begingroup$
    Good idea, but you rendered $2031^2030^··· bmod 125$. You need $2032^2031^2030^··· bmod 125$.
    $endgroup$
    – Oscar Lanzi
    Mar 25 at 1:30












    $begingroup$
    oops..............
    $endgroup$
    – fleablood
    Mar 25 at 11:58




    $begingroup$
    oops..............
    $endgroup$
    – fleablood
    Mar 25 at 11:58




    1




    1




    $begingroup$
    Actually I rendered it as $2032^2031^...equiv 31^2031^...pmod125$... which is still an error.
    $endgroup$
    – fleablood
    Mar 25 at 12:02




    $begingroup$
    Actually I rendered it as $2032^2031^...equiv 31^2031^...pmod125$... which is still an error.
    $endgroup$
    – fleablood
    Mar 25 at 12:02











    2












    $begingroup$

    By the Chinese Remainder Theorem, if you want to find what remainder a given number has when divided by $1000$, you can split that into 2 problems: Find the remainder$mod 8$ and$mod 125$. Obviously



    $$z_0:=2032^2031^2030^dots^2^1 equiv 0 pmod 8$$



    What remains to be found is $x_0 in [0,124]$ in



    $$z_0 equiv x_0 pmod 125.$$



    As $z_0$ is now coprime to $125$, you can apply Euler's theorem now. With



    $$z_1:=2031^2030^dots^2^1$$



    and



    $$phi(125)=100$$ the new problem becomes to find $x_1 in [0,99]$ in



    $$z_1 equiv x_1 pmod 100$$



    and then use



    $$32^x_1 equiv x_0 pmod 125$$



    to find $x_0$.



    So this reduced the original problem$mod 1000$ to a smaller problem$mod 100$.



    Applying this reduction procedure a few more times (using the Chinese Remainder Theorem if appropriate), should result in congruences with smaller and smaller module that can in the end be solved (e.g $mod 2$).



    Then to solve the original problem you need to back-substitute the calculated $x_i$ to get $x_i-1$, just as outlined for $x_1,x_0$ above.






    share|cite|improve this answer









    $endgroup$

















      2












      $begingroup$

      By the Chinese Remainder Theorem, if you want to find what remainder a given number has when divided by $1000$, you can split that into 2 problems: Find the remainder$mod 8$ and$mod 125$. Obviously



      $$z_0:=2032^2031^2030^dots^2^1 equiv 0 pmod 8$$



      What remains to be found is $x_0 in [0,124]$ in



      $$z_0 equiv x_0 pmod 125.$$



      As $z_0$ is now coprime to $125$, you can apply Euler's theorem now. With



      $$z_1:=2031^2030^dots^2^1$$



      and



      $$phi(125)=100$$ the new problem becomes to find $x_1 in [0,99]$ in



      $$z_1 equiv x_1 pmod 100$$



      and then use



      $$32^x_1 equiv x_0 pmod 125$$



      to find $x_0$.



      So this reduced the original problem$mod 1000$ to a smaller problem$mod 100$.



      Applying this reduction procedure a few more times (using the Chinese Remainder Theorem if appropriate), should result in congruences with smaller and smaller module that can in the end be solved (e.g $mod 2$).



      Then to solve the original problem you need to back-substitute the calculated $x_i$ to get $x_i-1$, just as outlined for $x_1,x_0$ above.






      share|cite|improve this answer









      $endgroup$















        2












        2








        2





        $begingroup$

        By the Chinese Remainder Theorem, if you want to find what remainder a given number has when divided by $1000$, you can split that into 2 problems: Find the remainder$mod 8$ and$mod 125$. Obviously



        $$z_0:=2032^2031^2030^dots^2^1 equiv 0 pmod 8$$



        What remains to be found is $x_0 in [0,124]$ in



        $$z_0 equiv x_0 pmod 125.$$



        As $z_0$ is now coprime to $125$, you can apply Euler's theorem now. With



        $$z_1:=2031^2030^dots^2^1$$



        and



        $$phi(125)=100$$ the new problem becomes to find $x_1 in [0,99]$ in



        $$z_1 equiv x_1 pmod 100$$



        and then use



        $$32^x_1 equiv x_0 pmod 125$$



        to find $x_0$.



        So this reduced the original problem$mod 1000$ to a smaller problem$mod 100$.



        Applying this reduction procedure a few more times (using the Chinese Remainder Theorem if appropriate), should result in congruences with smaller and smaller module that can in the end be solved (e.g $mod 2$).



        Then to solve the original problem you need to back-substitute the calculated $x_i$ to get $x_i-1$, just as outlined for $x_1,x_0$ above.






        share|cite|improve this answer









        $endgroup$



        By the Chinese Remainder Theorem, if you want to find what remainder a given number has when divided by $1000$, you can split that into 2 problems: Find the remainder$mod 8$ and$mod 125$. Obviously



        $$z_0:=2032^2031^2030^dots^2^1 equiv 0 pmod 8$$



        What remains to be found is $x_0 in [0,124]$ in



        $$z_0 equiv x_0 pmod 125.$$



        As $z_0$ is now coprime to $125$, you can apply Euler's theorem now. With



        $$z_1:=2031^2030^dots^2^1$$



        and



        $$phi(125)=100$$ the new problem becomes to find $x_1 in [0,99]$ in



        $$z_1 equiv x_1 pmod 100$$



        and then use



        $$32^x_1 equiv x_0 pmod 125$$



        to find $x_0$.



        So this reduced the original problem$mod 1000$ to a smaller problem$mod 100$.



        Applying this reduction procedure a few more times (using the Chinese Remainder Theorem if appropriate), should result in congruences with smaller and smaller module that can in the end be solved (e.g $mod 2$).



        Then to solve the original problem you need to back-substitute the calculated $x_i$ to get $x_i-1$, just as outlined for $x_1,x_0$ above.







        share|cite|improve this answer












        share|cite|improve this answer



        share|cite|improve this answer










        answered Mar 24 at 21:33









        IngixIngix

        6,8322510




        6,8322510



























            draft saved

            draft discarded
















































            Thanks for contributing an answer to Mathematics Stack Exchange!


            • Please be sure to answer the question. Provide details and share your research!

            But avoid


            • Asking for help, clarification, or responding to other answers.

            • Making statements based on opinion; back them up with references or personal experience.

            Use MathJax to format equations. MathJax reference.


            To learn more, see our tips on writing great answers.




            draft saved


            draft discarded














            StackExchange.ready(
            function ()
            StackExchange.openid.initPostLogin('.new-post-login', 'https%3a%2f%2fmath.stackexchange.com%2fquestions%2f3161051%2ffind-last-3-digits-of-203220312030-dots21%23new-answer', 'question_page');

            );

            Post as a guest















            Required, but never shown





















































            Required, but never shown














            Required, but never shown












            Required, but never shown







            Required, but never shown

































            Required, but never shown














            Required, but never shown












            Required, but never shown







            Required, but never shown







            Popular posts from this blog

            Kamusi Yaliyomo Aina za kamusi | Muundo wa kamusi | Faida za kamusi | Dhima ya picha katika kamusi | Marejeo | Tazama pia | Viungo vya nje | UrambazajiKuhusu kamusiGo-SwahiliWiki-KamusiKamusi ya Kiswahili na Kiingerezakuihariri na kuongeza habari

            Swift 4 - func physicsWorld not invoked on collision? The Next CEO of Stack OverflowHow to call Objective-C code from Swift#ifdef replacement in the Swift language@selector() in Swift?#pragma mark in Swift?Swift for loop: for index, element in array?dispatch_after - GCD in Swift?Swift Beta performance: sorting arraysSplit a String into an array in Swift?The use of Swift 3 @objc inference in Swift 4 mode is deprecated?How to optimize UITableViewCell, because my UITableView lags

            Access current req object everywhere in Node.js ExpressWhy are global variables considered bad practice? (node.js)Using req & res across functionsHow do I get the path to the current script with Node.js?What is Node.js' Connect, Express and “middleware”?Node.js w/ express error handling in callbackHow to access the GET parameters after “?” in Express?Modify Node.js req object parametersAccess “app” variable inside of ExpressJS/ConnectJS middleware?Node.js Express app - request objectAngular Http Module considered middleware?Session variables in ExpressJSAdd properties to the req object in expressjs with Typescript